Min and max eigenvalues of a quadratic form











up vote
0
down vote

favorite












I have the following quadratic form :
$q(x,y,z)=x(y+4z)+y(x-2z)+x^2$, represented by the symmetric matrix $begin{bmatrix}1&1&2\1&0&-1\2&-1&0end{bmatrix}$.
I am asked to find its signature, which appears to be $(2,1)$. And then, I am asked to prove that all of its matrix's eigenvalues are in $]-3,3[$ without computing them. I can't see how to do this.










share|cite|improve this question






















  • "...appears to be"? Can't you just calculate it? And what does "computing them" (the eigenvalues) mean? Without finding them explicitly but with the char. pol. it looks like a more or less easy exercise in Calculus I, IVT and etc.
    – DonAntonio
    Nov 14 at 10:35












  • Use Gershgorin circle theorem
    – Yadati Kiran
    Nov 14 at 10:46










  • @YadatiKiran Gershgorin's Theorem doesn't give so tight a bound for the eigenvalues of the matrix. As far as I can see it only gives $;(-3,4);$...
    – DonAntonio
    Nov 14 at 10:49










  • The bound is $(-2,4)$. We get three discs $D(1,3), D(0,2)$ and $D(0,3)$
    – Yadati Kiran
    Nov 14 at 11:00










  • We can also say $sum|lambda-a_{ii}|leqsum R_i implies |lambda-1|+|lambda|+|lambda|leq8 implies dfrac{-7}{3}leqlambdaleq3$
    – Yadati Kiran
    Nov 14 at 11:07

















up vote
0
down vote

favorite












I have the following quadratic form :
$q(x,y,z)=x(y+4z)+y(x-2z)+x^2$, represented by the symmetric matrix $begin{bmatrix}1&1&2\1&0&-1\2&-1&0end{bmatrix}$.
I am asked to find its signature, which appears to be $(2,1)$. And then, I am asked to prove that all of its matrix's eigenvalues are in $]-3,3[$ without computing them. I can't see how to do this.










share|cite|improve this question






















  • "...appears to be"? Can't you just calculate it? And what does "computing them" (the eigenvalues) mean? Without finding them explicitly but with the char. pol. it looks like a more or less easy exercise in Calculus I, IVT and etc.
    – DonAntonio
    Nov 14 at 10:35












  • Use Gershgorin circle theorem
    – Yadati Kiran
    Nov 14 at 10:46










  • @YadatiKiran Gershgorin's Theorem doesn't give so tight a bound for the eigenvalues of the matrix. As far as I can see it only gives $;(-3,4);$...
    – DonAntonio
    Nov 14 at 10:49










  • The bound is $(-2,4)$. We get three discs $D(1,3), D(0,2)$ and $D(0,3)$
    – Yadati Kiran
    Nov 14 at 11:00










  • We can also say $sum|lambda-a_{ii}|leqsum R_i implies |lambda-1|+|lambda|+|lambda|leq8 implies dfrac{-7}{3}leqlambdaleq3$
    – Yadati Kiran
    Nov 14 at 11:07















up vote
0
down vote

favorite









up vote
0
down vote

favorite











I have the following quadratic form :
$q(x,y,z)=x(y+4z)+y(x-2z)+x^2$, represented by the symmetric matrix $begin{bmatrix}1&1&2\1&0&-1\2&-1&0end{bmatrix}$.
I am asked to find its signature, which appears to be $(2,1)$. And then, I am asked to prove that all of its matrix's eigenvalues are in $]-3,3[$ without computing them. I can't see how to do this.










share|cite|improve this question













I have the following quadratic form :
$q(x,y,z)=x(y+4z)+y(x-2z)+x^2$, represented by the symmetric matrix $begin{bmatrix}1&1&2\1&0&-1\2&-1&0end{bmatrix}$.
I am asked to find its signature, which appears to be $(2,1)$. And then, I am asked to prove that all of its matrix's eigenvalues are in $]-3,3[$ without computing them. I can't see how to do this.







eigenvalues-eigenvectors quadratic-forms






share|cite|improve this question













share|cite|improve this question











share|cite|improve this question




share|cite|improve this question










asked Nov 14 at 10:30









James Well

511410




511410












  • "...appears to be"? Can't you just calculate it? And what does "computing them" (the eigenvalues) mean? Without finding them explicitly but with the char. pol. it looks like a more or less easy exercise in Calculus I, IVT and etc.
    – DonAntonio
    Nov 14 at 10:35












  • Use Gershgorin circle theorem
    – Yadati Kiran
    Nov 14 at 10:46










  • @YadatiKiran Gershgorin's Theorem doesn't give so tight a bound for the eigenvalues of the matrix. As far as I can see it only gives $;(-3,4);$...
    – DonAntonio
    Nov 14 at 10:49










  • The bound is $(-2,4)$. We get three discs $D(1,3), D(0,2)$ and $D(0,3)$
    – Yadati Kiran
    Nov 14 at 11:00










  • We can also say $sum|lambda-a_{ii}|leqsum R_i implies |lambda-1|+|lambda|+|lambda|leq8 implies dfrac{-7}{3}leqlambdaleq3$
    – Yadati Kiran
    Nov 14 at 11:07




















  • "...appears to be"? Can't you just calculate it? And what does "computing them" (the eigenvalues) mean? Without finding them explicitly but with the char. pol. it looks like a more or less easy exercise in Calculus I, IVT and etc.
    – DonAntonio
    Nov 14 at 10:35












  • Use Gershgorin circle theorem
    – Yadati Kiran
    Nov 14 at 10:46










  • @YadatiKiran Gershgorin's Theorem doesn't give so tight a bound for the eigenvalues of the matrix. As far as I can see it only gives $;(-3,4);$...
    – DonAntonio
    Nov 14 at 10:49










  • The bound is $(-2,4)$. We get three discs $D(1,3), D(0,2)$ and $D(0,3)$
    – Yadati Kiran
    Nov 14 at 11:00










  • We can also say $sum|lambda-a_{ii}|leqsum R_i implies |lambda-1|+|lambda|+|lambda|leq8 implies dfrac{-7}{3}leqlambdaleq3$
    – Yadati Kiran
    Nov 14 at 11:07


















"...appears to be"? Can't you just calculate it? And what does "computing them" (the eigenvalues) mean? Without finding them explicitly but with the char. pol. it looks like a more or less easy exercise in Calculus I, IVT and etc.
– DonAntonio
Nov 14 at 10:35






"...appears to be"? Can't you just calculate it? And what does "computing them" (the eigenvalues) mean? Without finding them explicitly but with the char. pol. it looks like a more or less easy exercise in Calculus I, IVT and etc.
– DonAntonio
Nov 14 at 10:35














Use Gershgorin circle theorem
– Yadati Kiran
Nov 14 at 10:46




Use Gershgorin circle theorem
– Yadati Kiran
Nov 14 at 10:46












@YadatiKiran Gershgorin's Theorem doesn't give so tight a bound for the eigenvalues of the matrix. As far as I can see it only gives $;(-3,4);$...
– DonAntonio
Nov 14 at 10:49




@YadatiKiran Gershgorin's Theorem doesn't give so tight a bound for the eigenvalues of the matrix. As far as I can see it only gives $;(-3,4);$...
– DonAntonio
Nov 14 at 10:49












The bound is $(-2,4)$. We get three discs $D(1,3), D(0,2)$ and $D(0,3)$
– Yadati Kiran
Nov 14 at 11:00




The bound is $(-2,4)$. We get three discs $D(1,3), D(0,2)$ and $D(0,3)$
– Yadati Kiran
Nov 14 at 11:00












We can also say $sum|lambda-a_{ii}|leqsum R_i implies |lambda-1|+|lambda|+|lambda|leq8 implies dfrac{-7}{3}leqlambdaleq3$
– Yadati Kiran
Nov 14 at 11:07






We can also say $sum|lambda-a_{ii}|leqsum R_i implies |lambda-1|+|lambda|+|lambda|leq8 implies dfrac{-7}{3}leqlambdaleq3$
– Yadati Kiran
Nov 14 at 11:07

















active

oldest

votes











Your Answer





StackExchange.ifUsing("editor", function () {
return StackExchange.using("mathjaxEditing", function () {
StackExchange.MarkdownEditor.creationCallbacks.add(function (editor, postfix) {
StackExchange.mathjaxEditing.prepareWmdForMathJax(editor, postfix, [["$", "$"], ["\\(","\\)"]]);
});
});
}, "mathjax-editing");

StackExchange.ready(function() {
var channelOptions = {
tags: "".split(" "),
id: "69"
};
initTagRenderer("".split(" "), "".split(" "), channelOptions);

StackExchange.using("externalEditor", function() {
// Have to fire editor after snippets, if snippets enabled
if (StackExchange.settings.snippets.snippetsEnabled) {
StackExchange.using("snippets", function() {
createEditor();
});
}
else {
createEditor();
}
});

function createEditor() {
StackExchange.prepareEditor({
heartbeatType: 'answer',
convertImagesToLinks: true,
noModals: true,
showLowRepImageUploadWarning: true,
reputationToPostImages: 10,
bindNavPrevention: true,
postfix: "",
imageUploader: {
brandingHtml: "Powered by u003ca class="icon-imgur-white" href="https://imgur.com/"u003eu003c/au003e",
contentPolicyHtml: "User contributions licensed under u003ca href="https://creativecommons.org/licenses/by-sa/3.0/"u003ecc by-sa 3.0 with attribution requiredu003c/au003e u003ca href="https://stackoverflow.com/legal/content-policy"u003e(content policy)u003c/au003e",
allowUrls: true
},
noCode: true, onDemand: true,
discardSelector: ".discard-answer"
,immediatelyShowMarkdownHelp:true
});


}
});














 

draft saved


draft discarded


















StackExchange.ready(
function () {
StackExchange.openid.initPostLogin('.new-post-login', 'https%3a%2f%2fmath.stackexchange.com%2fquestions%2f2998102%2fmin-and-max-eigenvalues-of-a-quadratic-form%23new-answer', 'question_page');
}
);

Post as a guest















Required, but never shown






























active

oldest

votes













active

oldest

votes









active

oldest

votes






active

oldest

votes
















 

draft saved


draft discarded



















































 


draft saved


draft discarded














StackExchange.ready(
function () {
StackExchange.openid.initPostLogin('.new-post-login', 'https%3a%2f%2fmath.stackexchange.com%2fquestions%2f2998102%2fmin-and-max-eigenvalues-of-a-quadratic-form%23new-answer', 'question_page');
}
);

Post as a guest















Required, but never shown





















































Required, but never shown














Required, but never shown












Required, but never shown







Required, but never shown

































Required, but never shown














Required, but never shown












Required, but never shown







Required, but never shown







Popular posts from this blog

Plaza Victoria

In PowerPoint, is there a keyboard shortcut for bulleted / numbered list?

How to put 3 figures in Latex with 2 figures side by side and 1 below these side by side images but in...